Assessment, Diagnosis and Management of Common Hematologic/ Endocrine Problems

Ace your homework & exams now with Quizwiz!

Jaundice 1. What is the difference between unconjugated and conjugated hyperbilirubinemia in young infants? 2. Discuss the common causes of each. 3. Describe the pathophysiology of jaundice. 4. Identify the common causes of jaundice in older children and adolescents. 5. What is the pertinent subjective and objective data you would obtain on a young infant presenting with jaundice? a). What laboratory tests would you request and why? 6. Discuss the differential diagnoses of jaundice. 7. What infants/children would you refer to a physician? Why? 8. Discuss how you would manage hyperbilirubinemia in a breast-fed infant. 9. Discuss the management for the other diagnoses considered in #6 above.

1 & 2. unconjugated bilirubin: indirect —0.4 to 0.8 (levels greater than 20 mg/dL may be neurotoxic to brain) may be caused by overproduction of bilirubin, impaired conjugation, transport or uptake of bilirubin a. Physiological hyperbilirubinemia b. Breastmilk jaundice c. Prolonged (nonphysiologic) neonatal hyperbilirubinemia—associated with breastfeeding, maternal diabetes, induced labor, prematurity, Asian ethnicity, and male gender congenital familial nonhemolytic jaundice conjugated bilirubin: direct —0.2 to 0.4 (higher levels require investigation for pathology) obtructive type caused by range of pathologic conditions (rare in newborns) including biliary obstruction, infection, drugs (aspirin, acetaminophen), and other metabolic disorders 3. from epocrates: Bilirubin is the final product of heme catabolism, which is mostly derived from breakdown of the hemoglobin. Heme can also be derived from breakdown of other proteins. heme is further catabolized by heme oxygenase to biliverdin. This is acted upon by biliverdin reductase to form bilirubin. Bilirubin is then bound to serum albumin in the plasma and is transported to the liver. Bilirubin dissociates from albumin, and with the help of carrier proteins such as ligandins, is taken up into the hepatocytes. Bilirubin is then conjugated by the UDPGT enzyme in the hepatocytes. Bilirubin glucuronide reaches the intestines via the gall bladder and common bile duct. In the intestines of the newborn, most of the bilirubin glucuronide is unconjugated by beta glucuronidase. Some of this unconjugated bilirubin is reabsorbed and gets into the enterohepatic circulation. The rest of the conjugated bilirubin reaches the colon, where the bacteria break it down to urobilinogen, which is then excreted. When this normal process of bilirubin formation and excretion is disrupted, hyperbilirubinemia results. 4. From epocrates: Alcoholic liver disease, Choledocholithiasis, Hepatitis E, *Hepatitis A, Hepatitis B*, Hepatitis C, Nonalcoholic steatohepatitis, Drug-induced direct hyperbilirubinemia, Ascending cholangitis, Autoimmune hepatitis, Pancreatic carcinoma, Hemochromatosis, *Pregnancy*, Postoperative stricture, Drug-induced indirect hyperbilirubinemia, *Hemolytic anemia*, Gilbert syndrome, Carotenemia 5. risk factors: • Significant hemolytic disease, anemia • Inborn errors of metabolism • Early or severe jaundice • Ethnic or geographic origin associated with hemolytic anemia • Hepatobiliary disease • Previous sibling received phototherapy History • ABO or Rh incompatibilities in previous pregnancies • Sepsis risk for the infant, such as prolonged rupture of maternal membranes • Macrosomic infant of a diabetic mother Physical Examination • Jaundice at birth or at any time during the neonatal period, with face affected first, followed by the shoulders, chest, and abdomen. Jaundice from deposition of indirect bilirubin in the skin tends to appear bright yellow or orange; jaundice of the obstructive type (direct bilirubin) appears greenish or muddy yellow, with the difference apparent only in severe jaundice. • A crude estimate of the level of jaundice can be based on the dermal zone in which the jaundice is noticed. can help determine whether acquiring a total serum bilirubin (TSB) or a transcutaneous bilirubin (TcB) is warranted. Head and neck—a mean bilirubin of 6 mg/dL Trunk and umbilicus—a mean bilirubin of 9 mg/dL Groin including the upper thighs—a mean bilirubin of 12 mg/dL Knees and elbows (including the ankles and wrists) or to the feet and hands (including the palms and soles)—a mean bilirubin of 15 mg/dL • Petechiae, bruising, hepatosplenomegaly, or signs of infection • Lethargy, hypotonia, poor feeding, and loss of the Moro reflex are common initial signs of bilirubin toxicity to the brain (kernicterus). These symptoms are subtle and indistinguishable from those of sepsis, asphyxia, hypoglycemia, intracranial hemorrhage, and other acute illnesses in the neonate. • Diminished tendon reflexes, respiratory distress, failure to suck, opisthotonos (spasm of the muscles causing backward arching of the head, neck, and spine, as in severe tetanus, some kinds of meningitis, and strychnine poisoning.), bulging fontanelle, twitching of face or limbs, seizures, and a shrill, high-pitched cry are later signs of kernicterus. Diagnostic Studies • TcB • TSB level (indirect and direct) for infants who have a TcB >15, for darker skinned infants, or for infants under phototherapy • If suspect that the total bilirubin is significantly elevated for the age of the infant, extra blood can be drawn and held for further testing, eliminating a return visit, stick, and/or unnecessary expense if all of the tests are not later indicated. Tests that may be indicated include: -ABO, Rh, blood type, isoimmune antibodies of mother (should be available at prenatal and delivering hospital), Coombs test on infant (many times this is done at delivery and held in the hospital's laboratory) -Hemoglobin, hematocrit, reticulocyte count 6. Elevated indirect (unconjugated) serum bilirubin with a normal reticulocyte count and negative Coombs test indicates conditions such as physiologic jaundice, breast milk jaundice, or congenital familial nonhemolytic jaundice. Elevated indirect serum bilirubin with an increased reticulocyte count indicates increased hemolysis secondary to conditions such as isoimmunization (positive Coombs test, such as caused by ABO or Rh incompatibility), abnormal red blood cell shape, or red blood cell enzyme abnormalities. Elevated indirect and direct serum bilirubin with a negative Coombs test and a normal reticulocyte count indicates hepatitis, metabolic abnormalities, biliary atresia, choledochal cyst (in the bile duct), gastrointestinal or pancreatic obstruction, sepsis, or drugs. Pathologic jaundice requires a more in-depth workup for the cause. Risk factors include: • Appearance of jaundice in first 24 hours of life • Rise of bilirubin greater than 0.5 mg/dL/hr • Conjugated bilirubin greater than 2 mg/dL physiologic jaundice - most common breast milk jaundice hyperbilirubinemia caused by metabolic factors: hypoxia, respiratory distress, lack of carbohydrates, hormonal influences, cretinism, genetic factors, Crigler-Najar syndrome, transient familial hyperbilirubinemia, vitamin K, novoblocin hemolytic states and hematoma: erythroblastosis: Rh and ABO, congenital hemolytic states: sphericity, nonspherocytic, infantile pyknocytosis, vitamin K, enclosed hemorrhage - hematoma mixed hemolytic and hepatotoxic factors: infection: bacterial sepsis, pyelonephritis, hepatitis, toxoplasmosis, cytomegalic inclusion disease, rubella, vitamin K hepatocellular damage: biliary atresia, galactosemia, hepatitis, infection Increased rate of hemolysis: ABO incompatibility, Rh incompatibility, abnormal red blood cell shapes (spherocytosis, elliptocytosis, pyknocytosis, and stomatocytosis), red blood cell enzyme abnormalities (glucose-6-phosphate dehydrogenase deficiency, pyruvate kinase deficiency) • Decreased rate of conjugation: Immaturity of bilirubin conjugation (physiologic jaundice), congenital familial nonhemolytic jaundice (inborn errors of metabolism affecting glucuronyl transferase system and bilirubin transport), breast milk jaundice • Abnormalities of excretion or absorption: Sepsis, hepatitis (viral, parasitic, bacterial, toxic), metabolic abnormalities (galactosemia, glycogen storage disease, IDM, cystic fibrosis), biliary atresia, choledochal cyst, obstruction of ampulla of Vater (annular pancreas), drugs 7. if infant has signs of underlying illness such as lethargy, apnea, temperature instability, behavior changes, hepatosplenomegaly, poor skin perfusion if infant <35 weeks gestational or 2500 grams Direct (conjugated) hyperbilirubinemia—treat underlying disease and/or refer as appropriate symptomatic infants, those with severe hyperbilirubinemia, and infants at risk for severe hyperbilirubinemia who fail to respond adequately to phototherapy total bilirubin levels >95th percentile in an hour-specific nomogram: no increased risk of kernicterus total bilirubin levels >95th percentile in an hour-specific nomogram: high risk of kernicterus with exchange transfusion and IVIG 8. Continue breastfeeding unless clinical signs of pathological jaundice are observed. The family should be reassured that breast milk jaundice is not harmful. Promote breastfeeding by advising mothers to put the baby to the breast 8 to 12 times per day for the first several days and discourage the use of routine supplementation of water or dextrose water. If the breastfeeding infant requires phototherapy, breastfeeding should be continued. It is also an option to temporarily interrupt breastfeeding (have the mother pump to maintain her supply) and substitute formula for 24 hours. In breastfed infants receiving phototherapy, supplementation with expressed breast milk or milk- based formula is appropriate if the infant's intake is inadequate, weight loss is excessive (greater than 10% of birthweight), or the infant seems dehydrated 9. • Phototherapy is used to treat elevated indirect hyperbilirubinemia. Home phototherapy can be used for those infants without risk factors Phototherapy is contraindicated with elevated direct bilirubin. infant dressed only in a diaper and should be wearing eye shields. Three types of phototherapy are used: -Banks of overhead lights placed close to the infant requires eyepatch removal at regular intervals, taking care to prevent corneal abrasions; monitoring of temperature; increased fluid intake in response to evaporative water losses; and avoidance of oral drugs because of decreased absorption -Biliblanket (fiberoptic pad) allows ongoing interaction between mother and infant -Bilibed • Rebound bilirubin testing (measurement of bilirubin after phototherapy is discontinued) is not required in full-term newborns with physiologic jaundice • exchange transfusion levels severe hyperbilirubinemia perform STAT labs obtain maternal blood type, Rh and ABO screen from hospital D/C summary Meausre infants total and direct serum bilirubin Other labs: Coombs, blood type, G6PD screen, CBC, hemoglobin electrophoresis, chemistry panel jaudince for <2-3 weeks: total and direct bill, check newborn screen for hypothyroidism and galactosemia ABO incomp: Monitor unconjugated bilirubin levels, Phototherapy if indicated based on bilirubin, gestational, and post-delivery age of neonate, If anemia is severe, may require transfusion Rh incompatibility: Mother: Rh isoimmunization screen at first prenatal visit. If mother Rh negative, test father; if father is Rh positive then the pregnancy is at risk. Risk for problems increases with each pregnancy as antibody levels rise. Administration of Rh immune globulin after any invasive procedure during pregnancy and after the termination of each pregnancy (including any miscarriage and/or abortion) Infant: Antenatal treatment—once diagnosis has been established, transfusion of fetus with Rh negative blood. Postpartum treatment (1) Phototherapy, with exchange transfusion if indicated by bilirubin level (2) Transfusion of packed red blood cells if indicated by hemoglobin level (3) efficacy of administration of gamma globulin hepatitis: Chronic infection with hepatitis B occurs if there is an absence of IgM anti-HBc or the persistence of HBsAg for 6 months. To try to avoid perinatal exposure—HBIG 0.5 mL vaccination in first 12 hours of life; the second dose must be given at 1 to 2 months with third dose at 6 months; the infant should have testing for HBsAg and anti-HBs at 9 to 18 months to measure success of vaccination with HBIG at birth 3. Prevention of hepatitis B is by universally vaccinating all children so that they cannot become carriers of hepatitis B Chronic infection with hepatitis B is being treated in adults with liver failure and chronic hepatitis C; there are 2 classes of agent interferons (interferon- 2b and peginterferon- 2a) or long-term use of nucleosides/nucleotide analogues (lamivudine, adefovir, entecavir, tenofovir, and telbivudine) being used as monotherapy or in combination Treatment is supportive, good nutrition, decreased activity, monitor hydration and chronic state 2. HAV—immunoglobulin (IG) available for decreasing course of disease in early stages or prevention in exposed individuals; not recommended for HCV or HEV; defer measles or MMR immunization for 3 months following administration of immune globulin 3. Treatment for HBV a. Interferon -2B treatment for chronic HBV; limited improvement d. Lamivudine 3 mg/kg/day may be used if interferon fails e. Adefovir 10 mg/day can be used in children 12 years of age 4. Treatment of HCV Interferon -2B, three times per week, plus oral ribavirin Pegylated interferon weekly, plus oral ribavirin, for children 3 years of age 5. Report to state health department 6. Prevention a. HAV—two inactive HAV vaccines recommended for children age 12 months; hepatitis A vaccine should be given to previously unvaccinated individuals prior to travel to affected areas; HAV vaccine can also be given as postexposure prophylaxis; for children aged 12 months, immunocompromised persons, persons with chronic liver c. disease, and persons who are allergic to the vaccine or a vaccine component HAV IG should be used c. Cautious intake of food and water when traveling to endemic area d. Avoid unprotected sex and drug use sepsis: Medical referral for hospitalization 2. Close observation, monitoring, and supportive care 3. IV fluids to maintain hemodynamics 4. Broad spectrum parenteral antibiotics for gram positive and negative organisms pending culture and sensitivities 5. Vasoactive medication if septic shock ensues 6. Decision making regarding age and relative risk a. Neonates 28 days with FWS 38°C, complete sepsis workup with CBC with differential, blood cultures, U/A and urine culture, CSF studies, and chest x-ray; requires hospitalization with IV antibiotics, pending culture results b. Infants 28 to 90 days with FWS 38°C can be managed as outpatient without antibiotics if all low-risk criteria are met; outpatient treatment for UTI if able to take oral meds; if not UTI and low-risk criteria not met, then admit for LP and IV antibiotics pending culture results c. Infants 90 days with FWS 39°C who have received HIB and PCV-7, require at a minimum a U/A and UC; for all females, uncircumcised males 24 months, circumcised males 6 months, and any infant with a previous UTI 7. Close follow-up, often seen outpatient within 24 hours after discharge from hospital pyelonephritis: antibiotics: bactrim, amoxicilin, augmentin, cephalexin, nitrofurantoin, Follow-up urine cultures a. Second culture at 72 hours after initiating treatment if symptoms are not resolving b. Culture one week after completion of treatment c. Close monitoring of periodic urine cultures with recurrent infections and/or unexplained fevers 3. Education/prevention a. Increased fluid intake b. Frequent voiding with complete emptying of bladder c. Good perineal hygiene with front-to-back wiping d. Avoid bubble baths and other urethral irritants toxoplasmosis: 1. Prevention is best—pregnant women should not eat raw or undercooked meat; they should wash fruits and vegetables prior to eating and should wear gloves while gardening, avoid changing cat litter 2. Congenital—pyrimethamine, sulfadiazine, and leucovorin is most common treatment for overt toxoplasmosis in newborn; if treated, ocular prognosis is better, but late onset visual problems can still occur a. Maternal treatment with pyrimethoamine and sulfadiazine b. Clindamycin as alternative if sulfa compound allergic c. Leucovorin is taken with pyrimethamine cytomegalic inclusion disease: Primary prevention in women of child-bearing age and younger, since both initial infection and reactivation during pregnancy can cause fetal infection Asymptomatic infants with congenital CMV a. Refer to audiology for periodic sensorineural hearing evaluation b. Screen frequently for growth retardation and emerging developmental delays c. Refer to ophthalmology—emerging chorioretinitis d. Refer to dentistry—defective tooth enamel 4. Neonatal acquisition—often presents as afebrile pneumonia after 8-week incubation period; test suspect infants 5. Seek specialist consultation regarding use of ganciclovir rubella: Management of uncomplicated infection is primarily supportive—includes fever and pain (in lymph nodes); control with acetaminophen or ibuprofen 2. Determine contacts that may require immunization Infected children should limit contact with susceptible persons, including women of childbearing age; out of school for 5 days after onset of rash 4. Educate adolescent females regarding teratogenic nature of rubella in pregnancy, resulting in congenital rubella syndrome—multiple congenital anomalies affecting eyes, heart, auditory with hearing loss, and neurologic systems 5. Educate caretakers regarding complications of arthritis, and rarely thrombocytopenia and encephalitis cystic fibrosis: Referral to cystic fibrosis care center for longterm care 2. Interdisciplinary management involving intensive education, airway clearance techniques, replacement of pancreatic enzymes, nutritional support, prevention and aggressive treatment of infection, and psychosocial support 3. Routinely given medications a. Inhaled mucolytic agent b. Recombinant human DNase (Pulmozyme) c. Inhaled tobramycin d. Oral azithromycin for chronic pseudomonas aeruginosa 4. Antibiotic therapy based on sputum culture and sensitivity a. Given early in course of disease to delay onset of chronic colonization with P. aeruginosa (the primary organism found in the airways of CF children and adults; once established, it is nearly impossible to eradicate) b. Once colonized, given to slow decline in pulmonary function c. Intravenous courses given for pulmonary exacerbations to restore pulmonary function and reduce symptoms 5. Isolation—standard precautions for all patients; contact and droplet precautions when indicated, minimize contact with other CF patients to avoid spread of infections (particularly if infected with Burkholderia cepacia, a virulent pathogen that has innate antibiotic resistance, causes rapid decline in lung function and is associated with mortality in CF patients) 6. Role of primary care provider—annual infl uenza vaccine, more liberal use of antibiotics for respiratory infections, be alert for CF complications

Mononucleosis 1. What is the etiology of infectious mononucleosis? 2. What is the epidemiology? 3. What are the most common clinical manifestations? 4. What differential diagnoses should be ruled out? 5. Discuss the lab and diagnostic tests that are performed. 6. Discuss the therapeutic management plan. 7. What anticipatory guidance should you provide for the patient and family? 8. Discuss when you should seek consultation from the physician?

1. caused by the Epstein-Barr family of herpesvirus in more than 90% of cases; the remaining cases are attributed to acute CMV, Toxoplasma gondii, adenovirus, viral hepatitis, HIV, and possibly rubella. The mode of transmission is personal contact, usually from deep kissing; by penetrative sexual contact; or from the exchange of saliva among children. The virus can live outside the body in saliva for several hours 2. Its distribution is worldwide, >95% having been infected. Older children and adolescents in poor urban settings or developing countries are seropositive for EBV. In these children primary infection tends to produce only mild symptoms and is subclinical. Exposure occurs in infancy or early childhood. In more developed countries infection in those <4 years of age is rare; 1/3 of cases occur during adolescence or young adulthood About 20% to 30% of healthy immune individuals shed EBV at any one time. From 60% to 90% of EBV-infected individuals on immunosuppressive therapy, including those on steroids. 3. disease of the primary lymphoid tissue and peripheral blood. Lymphoid tissue—regional lymph nodes, tonsils, spleen, and liver—is enlarged. Atypical lymphocytes are seen in the peripheral blood. Almost all body organs are involved, including but not limited to the lungs, heart, kidneys, adrenals, CNS, and skin. Symptoms are variable and can last up to 2 to 3 weeks. • Fever: Moderate to high fever (<103° F) is common (>90%). In severe cases, fever can reach 104° to 105° F. Fevers usually wane over a 10- to 14-day period. • Sore throat: Usually begins a few days after the fever with an average incidence of about 80%. The throat is very painful for 7 to 10 days. There is marked tonsillar enlargement, grayish exudates (in approximately 30% of cases), ulceration, and pseudomembrane formation. Petechiae are found on the palate (in approximately 25% to 60% of cases). The airway can be compromised (<5% of cases) and is an indication for hospitalization. • Lymphadenopathy: Anterior and posterior cervical and submandibular nodes are more commonly involved, less so the axillary and inguinal nodes. Epitrochlear lymphadenopathy is highly suggestive of the disease. Nodes are firm but usually nontender, are discrete, and range from 1 to 4 cm in size (up to 90% incidence). • Splenomegaly: Occurs typically 2 to 3 cm below the costal margin in approximately 50% of cases. Rupture is rare. • Hepatomegaly: 10% to 15%; jaundice occurs in about 5% of cases. • Skin rash: Occurs in 3% to 15% of cases, usually on the trunk, arms, and palms. It can be maculopapular, urticarial, scarlatiniform, hemorrhagic, or nodular (rarely petechial, vesicular, or hemorrhagic) and usually occurs during the first few days of symptomatology onset and lasts 1 to 6 days. The rash occurs more frequently in patients taking ampicillin (up to 100%); probably represents a form of arteritis or vasculitis rather than hypersensitivity to ampicillin, and typically starts 5 to 10 days after the drug has begun. A symmetric rash of erythematous papules with or without coalescence on the cheeks, extremities, and buttocks (looks like atopic dermatitis) is associated with EBV infection as well (the Gianotti-Crosti syndrome). • Vision: Perceptual distortions in size, shapes, and spatial relationships can occur. Periorbital edema has been reported in 30% of cases. Other systemic manifestations can include myalgia, arthralgia, headache, chest pain, nausea, anorexia, vomiting, ocular pain, photophobia, conjunctivitis, gingivitis, abdominal pain, orchitis (rare), diarrhea, cough, pneumonia, myocarditis, pericarditis, rhinitis, epistaxis, bradycardia, aseptic meningitis, GBS, Bell palsy, Reye syndrome, and acute cerebellar ataxia. 4. gram-positive alpha-beta hemolytic streptococcal pharyngitis, leukemia, lymphoreticular malignancies, adenoviruses, toxoplasmosis, CMV, rubella, HIV, hepatitis, SLE, drug reactions, and diphtheria. 5. >10% atypical lymphocytes and lymphocytosis; elevated liver enzymes are typical. Monospot and the serum heterophile test are positive in 85% of infected patients >4 years (often negative in those <4 years of age). Children >4 years usually must be ill for approximately 2 weeks before seroconverting. Viral culture and Epstein-Barr-specific core and capsule antibody testing are usually used for diagnosis if the primary screening test results are negative, and there is continued suspicion. Depending on the specific EBV antigen system tested, levels can be detectable for years after infection. 6. supportive with adequate bed rest (for debilitating cases), over-the-counter pain relievers, fluids, and calories. Corticosteroids and acyclovir are not recommended for routine uncomplicated disease; penicillin products should not be given. Contact sports and strenuous exercise should be avoided for 4 weeks and especially in those with hepatosplenomegaly. Symptoms generally resolve within 2 to 4 weeks; fatigue and weakness may persist for up to 6 to 12 months after severe infection. Complete recovery can be expected in more than 95% of cases without any specific treatment 7. Persons with a recent history of IMS or an infectious mononucleosis-like disease should not donate blood or organs. Contact sports and exercise - see above Recovery - see above avoid sharing drinks/utensils etc Advise the athlete to avoid any form of exertion, including all sports during the first 3 weeks (minimum) after onset of symptoms when the spleen is more likely to enlarge. • At 3 weeks after symptom onset, assuming the athlete is afebrile and symptom-free, he or she may return to light, noncontact activities. No sports should be played if there is risk of chest or abdominal contact or if it involves increased intraabdominal pressure or Valsalva maneuvers. • Fully returning to play should be made on a case-by-case basis and is generally considered safe at 4 weeks after symptom onset, assuming the patient's physical stamina has returned and all symptoms have resolved. If the sports involved increases intraabdominal pressure, a longer recovery time is suggested 8. Rare complications include splenic rupture, neurologic complications (from aseptic meningitis, encephalitis, myelitis, optic neuritis, cranial nerve palsies, GBS), thrombocytopenia, agranulocytosis, hemolytic anemia, orchitis, myocarditis, or chronic IMS. The virus also seems to increase the risk for Hodgkin disease.

Insulin Pump Management (ppt)

A few months later, you admit Daniel to the hospital for video EEG monitoring He is now on a Medtronic insulin pump Daniel's mother is going to stay with him and help to operate his pump and log his pump activity His initial blood glucose is 310 mg/dL, so you check his urine ketones and find them to be 2+. What now? a. Nothing. The pump will take care of it. b. He needs a dose of aspart, and there may be an issue with the pump which needs to be addressed. Insulin pumps are not foolproof and they require a significant amount of information and interaction to operate correctly insulin pump troubleshooting: A: Follow below steps to address the ketones Daniel should change his pump site Aspart to correct his hyperglycemia and ketosis Consider giving via injection in case there is any issue with the pump Recheck blood glucose and ketones after several hours If there is evidence of significant malfunction of the pump itself (as opposed to just the site), insulin can be given via injection, including glargine every 24 hours, until the issue is resolved or a replacement pump is received Rapid acting insulin runs continuously at a set rate in the pump to meet basal insulin requirements The basal rate can be set at different values throughout the day to match varying basal requirements during the day, eg 0.5 units/hour 6 am to 9 pm, 0.4 units/hour 9 pm to 3 am, 0.6 units/hour 3 am to 6 am Carbohydrate ratios, ISF and targets are also set in the pump. When Daniel enters the carbohydrate amount he plans to eat and/or his current blood glucose, the pump calculates an insulin dose to be given as a bolus based on these settings analogous to how he would have previously calculated an insulin dose to give as an injection. Insulin is delivered subcutaneously via a site which is changed every 2-3 days or sooner if unexpected hyperglycemia and/or ketosis develops which could suggest the site is not delivering insulin effectively

Daniel has new-onset Diabetes Mellitus (DM) (ppt)

Assess severity of his diabetes presentation with the following labs: Basic metabolic panel Venous blood gas Hemoglobin A1c β-hydroxybutyrate (serum ketones) Urinalysis for urine acetone (a.k.a urine ketones) Classify type of diabetes with these labs: Insulin level Prior to giving insulin C-peptide - Could still be sent even if exogenous insulin initiated Autoantibodies associated with type 1 DM such as: -GAD65 antibodies -Islet antigen-2 antibodies Note: C-peptide and insulin level should always be sent at same time as serum glucose Daniel remains on the insulin drip until diabetic ketoacidosis is resolved He now needs to transition to subcutaneous insulin What insulin regimen will you start him on? How will you treat hypo- and hyperglycemia? When will he be ready for discharge?

Hypoglycemia Management (ppt)

Daniel has a blood glucose of 52 before lunch He feels a bit shaky but is alert and answering questions His mother thinks he should get no aspart with lunch His father thinks he should get his usual dose of aspart Q: Who is right? a. His mother b. His father c. Neither Providing no insulin coverage when he eats will result in postprandial hyperglycemia Providing usual carbohydrate coverage should result in his blood glucose after the meal being similar to his glucose before the meal, so he will remain hypoglycemic if the hypoglycemia is not addressed first Correct: Neither is right Follow the "Rule of 15s" 15g of quick-acting carbohydrate e.g. 4oz of juice Recheck BG in 15 minutes Repeat this procedure until BG is >100 Daniel should then get his meal and apart according to the amount of carbohydrates he is eating •Glucagon should be given for severe hypoglycemia, ie when the patient is not fully alert and is unable to take a source of glucose by mouth •Hypoglycemia is the most common complication of type 1 diabetes

Diabetes Classification (ppt)

Daniel has type 1 DM and has little to no endogenous insulin (absolute deficiency) C-peptide/Insulin were low (with corresponding elevated serum glucose) Autoantibodies were elevated Family history was negative for diabetes, positive for celiac disease and autoimmune hypothyroidism (Hashimoto's thyroiditis) We must use exogenous insulin to function as his pancreas would Expected Daily Insulin Requirements (units of insulin/kg/day) relate to age and pubertal status, typical ranges shown below: Infants (0-2 yrs): <0.5 Pre-pubertal: 0.5-1 Adolescents (Pubertal): ~ 1 Estimated insulin requirement x patient's weight (kg) = Total Daily Dose (TDD) (units/day) The estimated TDD will be used to formulate an initial insulin regimen

Diabetes: Case Study (ppt)

Daniel is a 30 kg, pre-pubertal 9 year-old boy who presents to the emergency department with fatigue and vomiting He had been more thirsty and urinating more for 1 month. One day before admission, he began to complain of abdominal pain and began to vomit. He continued to vomit the next morning and was unable to hold down any fluid, so his mother brought him to the emergency department. In the emergency department, he was found to have a serum glucose of 545, pH 7.13, HCO3 7, β-hydroxybutyrate 5.2. He was started on a regular insulin drip at 0.1 units/kg/hr and admitted to the PICU.

Nutrition in Type 1 Diabetes (ppt)

Daniel is upset that he won't be able to have cake at his birthday party next month But he can! The dietician or other qualified healthcare worker should see him to teach Daniel and his family how to count carbohydrates He should avoid sugar sweetened beverages except to treat a low blood glucose Otherwise he should eat the same healthy, balanced diet as any child

Prescriptions for Home (ppt)

Long-acting insulin: Usually glargine Short-acting insulin: Lispro or aspart vials or pens Glucagon pens: 1 for home, 1 for school Pen needles if using pens Insulin syringes if using a vial Test strips and lancets -- Must be the versions that go with the provided glucometer -Brand based on insurance coverage Urine ketone strips

Type 2 DM (ppt)

Obesity -> Insulin resistance -> Relative Insulin Deficiency Signs of insulin resistance -Acanthosis nigricans -Skin tags Usually have negative autoantibodies and elevated C-peptide/insulin levels Less likely than type 1 to have DKA but can still occur Treatment: Lifestyle/metformin +/- insulin depending on severity

Sick Day Management (ppt)

One month later, you are working in the emergency room when Daniel comes in with his mother Daniel developed a sore throat and fever today His mother can't remember exactly how they are supposed to deal with illness now that he has diabetes Q: What do you tell them? Continue basal insulin regardless of PO intake An appropriate glargine dose will not cause hypoglycemia even if the patient is not eating at all Insulin requirements often increase during illness Monitor blood glucose and urine ketones frequently while ill If ketones present: Extra short-acting insulin is required to clear ketones Increased fluid intake also helps to clear ketones When else should you remind Daniel that he needs to check his ketones? When his BG is over a certain threshold, eg over 240

Basal Bolus Regimen (ppt)

TTD (wt x est. insulin req) 1/2 basal (long-acting) and 1/2 bolus (rapid acting -> carbohydrate coverage and hyperglycemia correction set dose - estimated by bolus insulin amount (1/2 TDD) divided by 3 OR carbohydrate ratio: 1 CR = 1 unit for every _ gram carbohydrates grams of carbohydrates eater x CR = insulin dose estimated by CR = 500 / TDD TDD: Total Daily Dose (estimated) Basal: Glargine, Detemir Bolus: Lispro, Aspart, Glulisine Carbohydrate coverage can be done in 2 ways: 1.Set dose •Units of insulin given for a meal, best done with set range of carbohydrate, eg 5 units with dinner which should be 45-60 grams carbohydrate 2.Carbohydrate ratio •Amount of insulin per grams of carbohydrate so that patient can adjust dose of insulin depending on what they choose to eat Hyperglycemia Correction: 1:ISF > Target Estimated by ISF = 1800 / TDD Approach to use of hyperglycemia correction at bedtime and overnight BG varies according to severity of hyperglycemia, presence of ketosis, age of patient, and knowledge of their sensitivity to insulin BG= blood glucose Target = BG over which correction is given ISF = Insulin Sensitivity Factor *Amount by which 1 unit of rapid-acting insulin will decrease BG Hyperglycemia correction is then expressed either as: 1. A correction factor 1:ISF > Target from which the patient can then calculate their correction by (BG - Target)/ISF = extra insulin needed 2. A sliding scale, eg if ISF = 50 and target = 150, then 1 unit for BG 151-200, 2 units for 201-250, etc *Daniel's regimen:* TDD = 20 U/d 29 kg x 0.7 units/d 1/2 basal = 10 units glargine (long acting) and 1/2 bolus = 10 units (rapid acting) Carbohydrate coverage: set carb amount 3 units/meal OR carbohydrate ratio: 1:25g, 500 / 20 = 25 (CR) Hyperglycemia correction: 1800/20 = 90 (ISF) BG - 200 (Target) / 90 1:90 > 200 TDD is 20 units/day 1. Basal: Glargine 10 units/day 2. Bolus: Aspart a. Carbohydrate coverage: 1:25 g b. Hyperglycemia correction (HC): 1 unit for every 90 that BG is > 200 Abbreviated as 1:90 >200 **Target BG is based on the ISF as well as age and reliability in reporting hypoglycemia. Example: If Daniel is eating 75 g of carbohydrate and his BG is 290, he will receive 4 units of aspart (3 for carbohydrate coverage plus 1 for hyperglycemia correction)

Ready for discharge (ppt)

What education does Daniel and his family need before he is discharged? Carbohydrate counting- often taught by a licensed dietician Diabetes education- often taught by a Certified diabetes educator (CDE) Anyone who will be involved in Daniel's care ideally should attend education sessions Consider meetings with a case manager and/or social worker to help Daniel's family prepare to care for him at home and school if these resources are available DISCHARGE CRITERIA Diabetes education completed Family feels comfortable with basic survival skills Dietician education completed Family feels comfortable with basic carbohydrate counting Prescriptions filled and brought to the hospital for review Follow-up appointment made Daniel's school notified

Adjusting the Regimen (ppt)

You ordered BG checks for Daniel before breakfast, lunch, and dinner, bedtime, and overnight as you do for all patients with diabetes Current regimen: glargine 10 units, apart cc 1:20g HC 1:90 >200 pre-breakfast HD2: bg: 132, carb amt 75g, aspart dose 3 units pre-lunch: bg: 252, carb amt: 50g, aspart: 3 units (2 CC + 1 HC) pre dinner: bg: 282, carb amt: 75 g, aspart: 4 units (3 CC + 1 HC) bedtime: bg: 292, carb: no food, aspart dose: none 2am: bg: 278, carb: no food, aspart: none pre breakfast hd 3: bg: 258, carb amt: 50g, aspart: 3 units (2 cc +1 hc) CC = carb coverage, HC = hyperglycemia correction Q: What would you like to change? a. Increase glargine dose b. Increase carbohydrate ratio Glargine should only be increased when the blood glucose is rising significantly overnight Correct: Change the carbohydrate ratio to 1:20 since the BG rose progressively with meals Determine which insulin dose is responsible for the out of range BGs, then make a 5-10% dose change BG rises > 50 points overnight -> Increase glargine Fasting hypoglycemia, BG falls > 50 overnight -> Decrease glargine Postprandial hypoglycemia -> Reduce carbohydrate ratio Postprandial hyperglycemia -> Increase carbohydrate ratio &/or increase hyperglycemia correction

Follow Up (ppt)

You see Daniel for a clinic visit 3 weeks later You review his blood glucoses and see that they were initially in the 200s after discharge, and the provider on call made several dose increases However, for the past week, his BGs have been mostly less than 100 and often less than 80. You decide to decrease both his glargine and his aspart doses Q: Why did his insulin requirements change? He does not actually have diabetes mellitus He has entered the "honeymoon" period He clearly met criteria for a diagnosis of diabetes mellitus. He is currently experiencing a known phenomenon that occurs after diagnosis. Correct: Insulin requirements decrease during: Honeymoon period: Recovery of remaining endogenous insulin production after initiating exogenous insulin therapy Insulin requirements increase during: DKA or recent resolution of DKA Illness Puberty Daniel has been getting all of his shots in his abdomen Q: What other sites can he use? Site rotation is important to prevent lipohypertrophy which interferes with insulin absorption upper arms, abdomen, hips and buttocks, front and outside side of thighs

Idiopathic Thrombocytopenic Purpura (ITP) a. Define each term b. Describe the pathophysiology c. List the clinical manifestations d. Discuss, briefly, the evaluation and management.

a. Antibody-Mediated Hemorrhagic Disease most common of the thrombocytopenic purpuras in childhood and is believed to be an autoimmune response in which circulating platelets are destroyed. usually occurs after viral illnesses many cases the cause is autoimmune. b. Secondary causes include leukemia, medications (e.g., quinine, heparin), lupus erythematosus, cirrhosis, HIV, hepatitis C, congenital causes, and von Willebrand factor deficiency. c. most symptoms do not develop until the platelet count is less than 20,000/mm3. • Acute onset of petechiae, purpura, and bleeding in an otherwise healthy child; the bruising or bleeding may be most prominent over the legs. • A viral illness 1 to 4 weeks before onset in 70% of cases. • Hemorrhage of the mucous membranes, particularly the gums and lips. • Nosebleeds that can be severe and difficult to control. • Menorrhagia in an adolescent female. • Liver, spleen, and lymph nodes are not generally enlarged. d. Laboratory findings in ITP include: • Low platelet count (less than 150,000/mm3) with an otherwise normal CBC • Normal PT and aPTT • Megathrombocytes on the peripheral smear • Normal WBC and RBC counts prognosis is excellent, with spontaneous recovery in 75% of pediatric cases in the first 3 months Most cases managed on outpatient basis without any specific therapy. If the platelet count is >50,000/mm3 and no bleeding is observed, children and parents should be advised to avoid contact sports, aspirin ingestion, and any other herbal or pharmacologic agents that interfere with platelet function and to notify the practitioner of any excessive bleeding. Epistaxis can be treated with local measures. In severe cases (platelets <50,000/mm3) after diagnosis of leukemia is ruled out, a short course of corticosteroid therapy may reduce severity in the initial phases. Intravenous immunoglobulin (IVIG) is also given to children with active severe bleeding and who have contraindications for steroid use; WinRho (Anti-D) is given intravenously with the dose depending on Hgb level; Rh(D) immune globulin is useful only in Rh-positive individuals. Splenectomy, immunosuppressives, and anti-CD20 antibody are options for those children with refractory or chronic ITP

Hemophilia a. Define each term b. Describe the pathophysiology c. List the clinical manifestations d. Discuss, briefly, the evaluation and management.

a. Inherited coagulation deficiencies are described according to the absent coagulation factor. Most result in abnormal bleeding. results from a deficiency of factor VIII (A) or factor IX (B). absence or deficiency of the coagulation factor results in prolonged bleeding either spontaneously from small vessels or as a result of trauma. b. to gauge the severity - the % of function of the factor levels with 100% (100 units/dL) equal to the function of factor found in 1 mL of normal plasma. The clotting factor levels associated with severity of bleeding are as follows: less than 1 unit/dL (<1%), severe; between 1 and 5 units/dL, moderate; and more than 5 units/dL, mild In plasma, factor VIII binds with von Willebrand factor (vWF), which is a specific circulatory protein and acts as a carrier protein. von Willebrand disease (also known as vascular hemophilia) is a heterogeneous group of hereditary bleeding disorders caused by a quantitative or qualitative abnormality of vWF protein. In type I the protein is quantitatively reduced; in type II it is qualitatively abnormal; and it is absent in type III. Because the genes for the coagulation factors are sex linked (carried on the X chromosome) and recessive, the disease affects primarily males. Females are generally only carriers of the disorder. Type A: x-linked factor VIII c. bleeding in muscle, joint, surgical d. PT - normal aPTT - prolonged bleeding time - normal Factor VIII coagulant activity (VIIC) - low Treatment: DDAVP or recombinent VIII Type B: x-linked factor IX c. bleeding in muscle, joint, surgical d. PT - normal aPTT - prolonged bleeding time - normal Factor IX: low Tx: Recombinant IX Type VW: autosomal dominant vWF and VIIC c. bleeding in mucous membranes, skin, surgical, menstrual d. PT - normal aPTT - prolonged or normal bleeding time - prolonged or normal VIIC: low or normal von Willebrand factor antigen (vWF: Ag): Low von Willebrand factor activity (vWF: Act): low Ristocetin-induced: normal, low or increased at low dose restocetin tx: DDAVP or vWF concentrate c. Hemophilia: • A positive family history in the vast majority of cases • Excessive bruising • Prolonged bleeding from mucous membranes after minor lacerations, immunizations, circumcision, or during menstruation (menorrhagia) • Hemarthrosis characterized by pain and swelling in the elbows, knees, and ankles • A greatly prolonged aPTT • A specific assay for factor VIII or IX activity confirms the diagnosis. von Willebrand disease: • Mucous membrane bleeding (epistaxis, menorrhagia), easy bruising, and excessive posttraumatic or postsurgical bleeding • History of ecchymosis of trunk, upper arms, and thighs • Factor VIII clotting activity usually decreased • vWF antigen usually decreased • Decreased vWF • Normal platelet count but isolated decreased platelet count associated with type 2B • Bleeding time and aPTT generally prolonged, but may be normal d. prevention of trauma and replacement therapy to increase factor VIII or factor IX activity in plasma. Plasma-derived and recombinant factor concentrates are available for replacement with recombinant factor preferred. Hemarthrosis is the leading type of significant local bleeding. application of cold and pressure to affected, painful joints. aspirin and NSAIDs avoided. Anticipatory guidance: avoiding high-risk behaviors and contact sports and wearing a bike helmet. Physical therapy may be needed to assist with decreased mobility caused by hemarthrosis and joint scarring. Psychosocial intervention may be needed to help families avoid overprotectiveness or permissiveness Ideally most children with hemophilia should be enrolled in a local hemophilia treatment center to facilitate a collaborative, interdisciplinary approach to management. The primary provider should remain central to the care of the child. All immunizations should be given subcutaneously with a 26-gauge needle, followed by firm pressure at the site for several minutes. Iron replacement may also be necessary in children with severe bleeding disorders. von Willebrand disease is treated depending on the type and severity of the bleeding. desmopressin (DDAVP) and factor VIII-vWF concentrates. Local measures to control bleeding may also be part of the treatment plan Adjunctive therapy (e.g., estrogen and/or aminocaproic acid) depends on the type of von Willebrand disease (type 1, 2A, 2B, 2M, 2N, or 3) which is determined by the level of qualitative or quantitative factor deficiency. The use of aminocaproic acid, an antifibrinolytic agent, is sometimes recommended for dental extraction and nosebleeds A written treatment plan tailoring replacement product dosage based on the location of the bleed should be in the chart and given to the parents to carry with them. The child should wear a Medic-Alert bracelet or necklace.

Lymphomas a. Define each term b. Describe the pathophysiology c. List the clinical manifestations d. Discuss, briefly, the evaluation and management.

a. Non Hodgkin Lymphoma: a diverse group of solid tumors of the lymphatic tissues that form from malignant proliferation of T cells, B cells, or indeterminate lymphocyte cells. common types are small noncleaved cell lymphoma (Burkitt and non-Burkitt subtypes, B-cell origin), lymphoblastic lymphoma, and large cell lymphoma b. most frequent malignancy in children with acquired immunodeficiency syndrome and is also associated with Epstein-Barr and cytomegalovirus infections. c. most common site of origin is in the lymphoid structures of the intestinal tract. The most common manifestations in children are (1) acute abdomen, including abdominal pain, distention, fullness, and constipation and (2) nontender lymph node enlargement. lymphoblastic often present as intrathoracic tumors; small noncleaved cell lymphomas commonly present as abdominal tumors. Other sites include the CNS and the bone marrow. Initial presentation is often with advanced disease of stages III or IV. Duration of symptoms before a diagnosis is made is typically 1 month or less d. CBC with differential, chest radiograph, ultrasound, CT or MRI scan or PET of the area in question, gallium and/or bone scan, bone marrow aspirates and biopsies, lumbar puncture with CNS fluid analysis, CBC, liver function tests, lactate dehydrogenase, uric acid and electrolyte levels, and 8-hour creatinine clearance if indicated The diagnosis is confirmed by surgical biopsy, and the extent of the disease process and staging can be determined by scans, bone marrow aspiration, and lumbar puncture. refer to a major pediatric cancer center for care. Lymphomas are sensitive to chemotherapy. Cranial irradiation or intrathecal chemotherapy is part of the treatment plan if CNS involvement is present. Maintenance therapy may be continued for 6 months to 2 years. The 5-year relative survival rate is 81.7%. a. Hodgkin Disease: malignancy of the reticuloendothelial and lymphatic systems and involves B cells. It usually originates in a cervical lymph node and spreads to other lymph node regions and, if left untreated, to organ systems, including liver, spleen, bone, bone marrow, and brain. involvement of the bone marrow and CNS is rare. Clinical and pathologic staging of the disease is usually done according to the Ann Arbor staging criteria. b. Clusters of cases in families suggest a genetic predisposition. Hodgkin disease is associated with immunodeficiencies and infection with Epstein-Barr virus (EBV) and cytomegalovirus virus. c. • Painless enlargement of the lymph nodes, usually in the cervical area; the nodes may feel firm, are often matted together, and are nontender to palpation • Chronic cough if the trachea is compressed by a large mediastinal mass • Fever, decreased appetite, weight loss, and night sweats d. Hematologic findings are often normal, but may include the following: • Anemia • Elevated or depressed leukocytes or platelets • Elevated sedimentation rate and C-reactive protein; serum copper and ferritin level • Abnormal liver function test results • Urinalysis may have proteinuria • Imaging studies: chest radiography, ultrasound, CT, MRI, and PET The diagnosis is confirmed by histologic examination of an excised lymph node, followed by bone marrow studies to determine the extent of the disease. receive treatment at a pediatric oncology center in collaboration with the primary provider. Optimal results are obtained through irradiation, chemotherapy with numerous agents or a combination of both. 95.1% relative survival rate Infertility is problematic for those receiving high doses of alkylators; sperm banking is discussed as an option for males before starting therapy. 30% of children who survive Hodgkin disease develop secondary malignancy 30 years later, typically thyroid, breast, nonmelanoma skin cancers and non-Hodgkin lymphoma and acute leukemia.

Thalassemias a. Define each term b. Describe the pathophysiology c. List the clinical manifestations d. Discuss, briefly, the evaluation and management.

a. a group of hereditary, hypochromic anemias that are associated with the absence or decreased synthesis of the normal Hgb polypeptide chains—usually the α- and ß-globin chains—and a relative excess of the other chains. The protein abnormality results in hemoglobinopathies whose names are based on the altered globulin chain. The possibility increases if the onset of anemia and symptoms is prior to 3 to 6 months of age and there is a family history of anemia, miscarriage, jaundice, gallstones, anemia, or splenomegaly. b. the heterozygous disease is hypochromic and microcytic, the homozygous diseases are also hemolytic. There is anemia and increased erythropoiesis. The erythropoiesis results in bone marrow expansion. Focal osteomalacia and delayed bone maturation are at least partially explained by suboptimal blood transfusions and iron overload. The effect on long-bone architecture makes the child more susceptible to fractures. the marrow expansion results in frontal bossing and hyperplasia of the maxillary bones leading to typical facies. Alpha composed of several variant hemoglobins subtypes - number of gene deletions of α-globin, with severity of symptoms increasing with more deletions. Three gene deletions result in severe, even fatal manifestation of disease. Two gene deletions present with hypochromia; the absence of gene deletions causes mild anemia and often erythrocytosis. A single globin gene deletion is clinically insignificant two manifestations: homozygous Hgb type, Barts hemoglobin with four γ- chains, results in hydrops fetalis and is incompatible with life because of severe anemia. homozygous Hgb H disease is a microcytic, hypochromic anemia that most often manifests as a hemolytic anemia, hepatosplenomegaly, and mild jaundice, and sometimes includes thalassemia-like bone changes. During times of physiologic stress the child may require RBC transfusion. two different carrier states: trait the child exhibits microcytosis and hypochromia, but has normal percentages of Hgb A2 and Hgb F. silent carrier state, but can have either a silent hematologic phenotype or present with microcytic hypochromia and some erythropoiesis. d. Hgb H disease exacerbations may necessitate occasional transfusion during hemolytic or aplastic crises. No treatment is indicated for the carrier trait expressions of disease. Beta Thalassemia Minor/Minima a. trait, associated with a mild, hypochromic, microcytic anemia in which Hgb levels are 2 to 3 g/dL below normal, and the MCV averages 65 fL. need to be monitored for iron accumulation but are otherwise asymptomatic may be confused with iron deficiency or lead poisoning and can be differentiated by measuring serum iron or lead levels, transferrin saturation, or serum ferritin levels avoid unnecessary administration of iron supplements that could result in iron overload. The primary diagnostic feature is increased Hgb A2 (greater than 3.5%) on electrophoresis. c. asymptomatic, although mild pallor and splenomegaly may be found. An Hgb of 9.5 to 11 g/dL and an MCV <80 fL/cell is commonly seen in prepubertal children. The MCV/RBC count per milliliter is <13 (the Mentzer Index). the Mentzer Index of iron deficiency is usually >13 The degree of anemia may be exacerbated in concurrent illness or pregnancy. d. No specific treatment is known. Primary emphasis should be on education of all family members and genetic testing, and counseling should be offered. Beta Thalassemia, Intermedia a. result of various mutations that cause a disorder with a clinical severity that spans from the mild symptoms of the beta-thalassemia trait to the severe manifestations of beta-thalassemia major. Classification is typically based on the severity of the symptoms and the types of treatments necessary rather than by the specific genotype. Diagnosis and management are clinically based with a goal toward maintaining a satisfactory Hgb of at least 6 to 7 g/dL without the regular need for RBC transfusions. Beta Thalassemia Major a. Homozygous forms are thalassemia intermedia and thalassemia major. Homozygous beta-thalassemia major (or Cooley anemia) is associated with severe anemia resulting from decreased or absent production of Hgb A and hemolysis caused by the precipitation of excess α-chains in the RBCs. c. Affected infants usually become symptomatic in the first year of life and have pallor, failure to thrive, hepatosplenomegaly, and a severe anemia with an average Hgb of 6 g/dL and low MCV (60 to 70 fL). RBC morphology reveals significant microcytosis, poikilocytosis, hypochromia, target cells, and nucleated RBCs. Hgb A2 and Hgb F levels are elevated. d. collaboration with a pediatric hematologist. Red blood cell transfusions are usually necessary every 2 to 4 weeks to maintain a hemoglobin level between 9 and 10 g/dL as the goal. To help with future crossmatching, the provider should obtain a complete typing of the patient's erythrocyte profile before the first transfusion. This helps decrease difficulties with subsequent transfusions. Splenectomy may be indicated as well. Stem cell transplantation is effective for the small percentage of young children with a human leukocyte antigen (HLA) match and no organ dysfunction. Allogeneic hematopoietic transplantation has been documented to produce approximately an 85% to 87% long-term survival rate in children. Gene therapy is being investigated and holds promise for those with this major disorder. Iron chelation is necessary to treat the hyperferric state produced by repeated transfusions and prevent complications primarily of the heart, liver, and endocrine system. Chronic iron chelation therapy is necessary in order to remove the excess iron that results from the frequent transfusions. Deferoxamine is administered parenterally usually via a pump overnight, associated with abdominal pain. Deferasirox is an oral agent, taken once daily, at a dose of 20 mg/kg/day; it stabilizes the ferritin levels, thus achieving a negative iron balance. Iron excretion through chelation is further aided by the ingestion of vitamin C. The iron is excreted through the kidneys, so hydration and monitoring of renal status is vital.

Leukemia a. Define each term b. Describe the pathophysiology c. List the clinical manifestations d. Discuss, briefly, the evaluation and management.

a. a group of malignant hematologic diseases in which normal bone marrow elements are replaced by abnormal, poorly differentiated lymphocytes known as blast cells. Genetic abnormalities in the hematopoietic cells take over and result in unregulated clonal proliferation of malignant cells classified according to cell type involvement (i.e., lymphocytic or nonlymphocytic) and by cellular differentiation. ALL is characterized by preponderantly undifferentiated WBCs. b. the exact cause of leukemia is unknown. Several factors associated with increased risk have been identified, including infection, radiation, chemical and drug exposure, and genetic factors (e.g., Down syndrome). c. clinical signs and symptoms are related to leukemic replacement of the bone marrow and the absence of blood cell precursors. anemic, pale, listless, irritable, or chronically tired and have the following: • A history of repeated infections, fever, weight loss • Bleeding episodes characterized by epistaxis, petechiae, and hematomas • Lymphadenopathy and hepatosplenomegaly • Bone and joint pain CNS symptoms are rare at the time of diagnosis but can present due to an intracranial or spinal mass. All these symptoms may be vague or nonspecific, in which case it is important for the provider to have a high index of suspicion for cancer. d. • CBC with differential WBC, platelet, and reticulocyte counts. Thrombocytopenia is present in up to 85% of cases, and anemia is also usually present. WBC count may be elevated, normal, or low with varying levels of neutropenia. • Peripheral smear may demonstrate malignant cells. • Bone marrow examination shows an infiltration of blast cells replacing normal elements of the marrow. 90% of children with ALL have genetic alterations in their leukemic blast cells including changes in the number of chromosomes and their structure with recurrent translocations noted in about half of the cases. Further classification regarding cell type, morphologic characteristics, and cell surface markers done at the cancer treatment center. 75% to 80% curable. Key genetic features are critical factors. The treatment program for most types of involves a 28-day induction phase (usually with vincristine, prednisone, and l-asparaginase), with the goal of inducing a complete remission and restoring normal hematopoiesis. This is followed by a consolidation phase of therapy and then a maintenance phase of therapy. Chemotherapy, CNS therapy (cranial irradiation, which is now reserved only for a high-risk child—those with CNS disease or high WBC counts at diagnosis, or intrathecal administration of chemotherapy), and systemic administration of corticosteroids are the key interventions. The use of cranial irradiation as part of therapy is decreasing. For children with ALL who relapse, the need for allogeneic stem cell transplantation is not considered until the second complete remission. children with the Philadelphia chromosome or those who are not in remission by the end of the first induction phases are considered high risk for relapse and need to consider transplantation sooner. For those with AML, allogeneic stem cell transplantation from an HLA-matched sibling or parent is considered in the first complete remission Long-term sequelae of cancer therapy for ALL include effects on cognition and neuropsychological functioning. CNS irradiation has been linked to learning disabilities and impaired IQ, especially in children younger than 5 years of age who also received intrathecal therapy. cranial radiation dosages have been reduced, and earlier neuropsychological testing is recommended. Other potential late effects of ALL treatment include congestive heart failure, avascular necrosis, and osteoporosis. TEL/AML1 is present in about 20% of children with ALL and is considered the most favorable prognostic marker; the presence of the Philadelphia chromosome is associated with a poor prognosis. role of the primary care provider - facilitate proper referrals and effective interdisciplinary communication and to assist the family in their coping and adaptation processes. Regular child health supervision visits are also important and should not be overlooked. The provider should ensure that immunizations are given as appropriate and that the child is monitored for failed remission or metastasis (CNS and testicles are common sites) and late effects.

Sickle Cell Disease a. Define each term b. Describe the pathophysiology c. List the clinical manifestations d. Discuss, briefly, the evaluation and management.

a. describes a group of complex, chronic disorders that are characterized by hemolysis, unpredictable acute complications that may become life threatening, and the possible development of chronic organ damage. autosomal recessive inheritance pattern. mostly african descent b. Children who have homozygous inheritance have SCA or disease (Hgb SS). Their bodies do not form the normal Hgb A molecule, but rather synthesize hemoglobin S (Hgb S), which carries the amino acid valine instead of glutamic acid. Hgb S tends to polymerize or come out of solution at low PaO2, low pH, low temperature, and low osmolality. This process collapses the RBC giving it a "sickled" shape and produces a chronic hemolytic anemia. The new shape is rigid and clogs small blood vessels producing ischemia, pain, and other vaso-occlusive problems. c. Children with sickle cell trait who are heterozygous (Hgb A + Hgb S) for the gene have a benign clinical course. Their RBCs contain only 30% to 40% Hgb S, and sickling does not occur under most conditions. It is only in rare instances of hypoxia, such as in shock, while flying in unpressurized aircraft, or traveling to high elevations, that signs of vaso-occlusion can occur. the presence of sickle cell trait has been implicated as a causative factor in the sudden deaths of young military recruits, college football players, and some teens. Extreme exercise, typically to exhaustion, dehydration, and relative hypoxia (altitude) are major confounding factors The symptoms of sickle cell disease are multisystem, necessitating vigilant care to minimize occurrence of crises and complications. Common symptoms include: • Fatigue and anemia • Pain crises • Dactylitis (swelling and inflammation of the hands and/or feet) and arthritis • Bacterial infections • Lung and heart injury • Leg ulcers • Priapism • Splenic sequestration (sudden pooling of blood in the spleen) and liver congestion • Aseptic necrosis and bone infarcts (death of portions of bone) • Eye damage • Abdominal pain Physical Examination Symptoms typically begin to emerge in the second 6 months of life as the amount of Hgb S increases and Hgb F declines. Subsequently, painful, vaso-occlusive crises occur. After 5 years old, splenomegaly usually disappears because of autoinfarction of the organ. Rates of height and weight gain usually slow after 7 years old, and puberty may be delayed 3 to 4 years. d. The following laboratory results are seen in sickle cell disease: • Hematocrit of 20% to 29% • Hgb 6 to 10 g/dL (severe) • Reticulocyte count elevated: 5% to 15% • Normal to increased WBC and platelet count • MCV >80 fL; mean corpuscular hemoglobin concentration (MCHC) >37 mg/dL • Hgb electrophoresis (after infancy), isoelectric focusing or high performance of liquid chromatography showing a predominance of Hgb S and no Hgb A. • Morphology: Irreversibly sickled cells or chronic elliptocytes, Howell-Jolly bodies, nucleated RBCs Hgb electrophoresis results in a newborn with sickle cell trait will be Hgb FAS, and Hgb FS for a child with either SCA or sickle beta-zero thalassemia (SBO). Normal results of Hgb electrophoresis are Hgb FA. Management done in consultation with a pediatric hematologist. still need regular primary care services. Growth is closely monitored, immunizations need to be done on time, annual ophthalmologic examination by a retinal specialist. Some of the key aspects of care for the child with SCA are as follows: • Hydration, illness prevention, and pain management are fundamental aspects of disease management. NSAIDs or acetaminophen may be adequate for mild to moderate pain, but narcotics should be used when these are not adequate for management • CBC and reticulocyte count are monitored every few months. • All the usual immunizations of childhood are to be administered on time including 13-valent pneumococcal conjugate and 23-valent pneumococcal polysaccharide vaccines. Meningococcal vaccine is administered for children older than 2 years; however, under special circumstances MPSV4 may be given to children as young as 3 months. An annual flu vaccination is essential. • Invasive bacterial infection is the leading cause of death in young children with SCA. Penicillin V prophylaxis (125 mg orally, twice daily) is initiated by 2 months old. At 3 years old, increase the dose to 250 mg orally twice a day, and continue at least until the fifth birthday • Folic acid supplementation at 1 mg/day is typically given to adults to prevent folate deficiency due to hemolysis. It is not standard therapy for children unless a folic acid deficiency is suspected • Aggressive treatment of infections and maintenance of hydration and body temperature are used to prevent hypoxia and acidosis; volume replacement may be necessary to prevent circulatory collapse. • Treatment of coexisting medical problems associated with lower O2 saturations, such as asthma and obstructive sleep apnea. • In children with severe SCA, hydroxyurea is used to reduce the number of painful crises and incidences of acute chest syndrome (a leading cause of death in adolescents with SCA). It is a preventive medication and not effective during the acute crisis. Hydroxyurea use is associated with a lower need of blood transfusions and fewer hospital visits. There is some early evidence suggesting it helps improve growth and preserves organ function. side effects do occur including increased risk for serious infection. • Annual stroke prevention screening of major intracranial vessels with transcranial Doppler ultrasound evaluation is planned for 2- to 16- year-old children. A reading of greater than 200 cm/sec time-averaged mean maximal velocity indicates high risk for stroke Emergency admission or referral is necessary in the presence of the following: • Fever (to rule out sepsis) greater than 101° F (38.3° C) • Pneumonia, chest pain, or other pulmonary symptoms (acute chest syndrome) • Sequestration crisis (splenomegaly with decreased Hgb or Hct) • Aplastic crisis (decreased Hct and reticulocyte count) • Severe painful crisis • Unusual headache, visual disturbances • Priapism • Consultation is also necessary for the chronic sequelae of persistent bone pain or leg ulcers, pregnancy, and contraception. Stem cell transplantation may be a consideration in children with significant disease and is curative in some persons. Gene therapy & new meds under investigation and may be available in the future.

Hereditary Spherocytosis a. Define each term b. Describe the pathophysiology c. List the clinical manifestations d. Discuss, briefly, the evaluation and management.

a. inherited, membranopathy hemolytic anemia b. characterized by a deficiency or abnormality of the RBC membrane protein spectrin, which reduces the RBC surface area. The RBC membranes assume a more spherical shape. Hence RBCs are more likely to be sequestered and prematurely destroyed in the spleen. causes mild chronic hemolysis to severe transfusion- dependent anemia c. none, family hx, hx of neonatal jaundice Jaundice usually appears in the newborn period, and it may be difficult to differentiate from hyperbilirubinemia caused by ABO incompatibility. After 2 years of age splenomegaly is usually present. Chronic fatigue, malaise, pallor, and abdominal pain may also be noted. d. spherocytosis Coombs' test negative Reticulocyte % increased, ranges from 5% to 20%. Osmotic fragility increased MCHC increased Chronic anemia (Hgb is 6 to 10 g/dL) • On peripheral smear, a small proportion of the RBCs is spherocytic and smaller than normal and lacks the central pallor of the usual biconcave disk-shaped cell. increased rate of autohemolysis of incubated blood. No splenectomy if Hgb >10 and reticulocyte <10% Folic acid (0.5 mg daily <5yr old, 1mg daily >5yr old) Splenectomy; immunizations for pneuomoccus, Hib, and meningococcus prior, penicillin prophylaxis severe, requiring multiple transfusions -> splenectomy, which usually produces a clinical cure. deferred until 5 or 6 years of age because of the increased risk of encapsulated bacterial infection before that age. Risks associated with splenectomy are postsplenectomy sepsis, penicillin-resistant pneumococci infection, pulmonary hypertension, and ischemic heart disease and stroke After splenectomy, prophylactic penicillin therapy (<5y: 125 mg orally twice a day; >5y: 250 mg orally twice a day) through adulthood is recommended. Because of increased hemolysis, children with HS and active hemolysis should receive 1 mg of folic acid daily until splenectomy. Splenectomy is an effective strategy to eliminate most of the hemolysis associated with HS

Hemolytic Disease of the Newborn a. Define each term b. Describe the pathophysiology c. List the clinical manifestations d. Discuss, briefly, the evaluation and management.

a. less than normal range of hemoglobin for birthweight and postnatal age. occurs secondary to acute blood loss before or during delivery. b. Acute blood loss after delivery can be external (gastrointestinal, circumcision site, umbilical stump), internal (fracture site, cephalhematoma, pulmonary hemorrhage, injured internal organ), or secondary to hemolysis or congenital aplastic or hypoplastic anemia. c. Twin-twin transfusion • Unexpected tearing or delayed clamping of umbilical cord resulting in neonatal blood loss • Internal hemorrhage (caused by fracture, cephalhematoma, or internal organ trauma) • Umbilical stump or circumcision bleeding • Pallor, congestive heart failure, and shock possible d. Treatment depends on cause. An asymptomatic full-term infant with a Hgb of 10 g/dL can be observed, whereas a symptomatic neonate born after abruptio placentae or with severe hemolytic disease of the newborn warrants transfusion. Treatment with blood should be balanced by concern about transfusion-acquired infection with cytomegalovirus (CMV), HIV, and hepatitis B and C viruses. Prognosis depends on the cause and severity of the anemia.

Iron Deficiency Anemia a. Define each term b. Describe the pathophysiology c. List the clinical manifestations d. Discuss, briefly, the evaluation and management.

acquired a. microcytic (small cell) and hypochromic (pale), infant and toddler, most common in children, decreased iron delivery to the marrow. caused by inadequate supply of iron, associated with low reticulocyte count and elevated red cell distribution width (RDW) AAP recommends universal Hgb screening for anemia at 12 months. include an assessment of risk factors: (1) history of prematurity or low birthweight; (2) exposure to lead; (3) exclusive breastfeeding beyond 4 months without iron supplementation; (4) weaning to whole cow's milk without iron sources; (5) feeding problems; (6) special health care needs; (7) low socioeconomic status; (8) Mexican-American descent. If at risk, a repeat Hgb/Hct should be performed as often as indicated. Screening can be performed on children < 1 when risk factors warrant it. Menstruating females may also require screening for IDA due to the monthly blood loss, rapid growth, and potentially inadequate diet. risk assessment should be performed at all preventative pediatric health visits b. excessive cow's milk ingestion/cow's milk protein intolerance, poor solid food intake, chronic blood loss, menstruation. result from a reduction in available Hgb for nutrient transport within the RBC. reduction of circulating Hgb results in small, pale RBCs (microcytic, hypochromic) with decreased oxygen-carrying potential. the presence of low levels of iron facilitates the intestinal absorption of lead c. Mild anemia—usually asymptomatic more severe anemia: waxy, sallow appearance of the skin, fatigue, irritability, pallor or none, restless, eating nonnutrient substances such as ice, plaster, clay, paint, fabric (pica), anorexia, developmental delays (mental and motor areas), behavioral disturbances, cognitive impairment rare: tachycardia, systolic murmurs, signs of congestive failure, hepatomegaly d. two most commonly used screening tests are Hgb and Hct Hgb: 8-11 (moderate), <7 (severe), MCV <60 (decreased), Retic: decreased to slightly increased, Serum Fe: decreased, TIBC: increased, % saturation decreased, serum ferritin: decreased, blood in stool or urine, ratio of MCV/RBC >13 High RDW (>14%) Mentzer Index >14 can also test transferrin and CRP severe anemia: stool guaiac testing mild to mod: Hgb 7-10 <4 - consult with hematologist <7 - referral to hematology red blood cell distribution width is the earliest marker ferrous sulfate: 4-6 mg/kg/day (or up to 6mg/kg/day) of elemental iron in three divided doses or 3mg/kg/day in 1-2 divided doses for mild or moderate, discontinue cow's milk, limit formula to <24oz/day and encourage solid food, correct diet, eliminate source of bleeding if age of child and dietary patterns consistent, begin trial of iron supplementation without further dx testing and then follow the child's Hgb and reticulocyte counts if tx is effective, follow up Hgb/hct and reticulocyte count in 1 month should reveal a 1g/dL improvement if adequate response - dx is confirmed peripheral reticulocytosis may be seen after the first days of tx, and Hgb should return to normal within 4-6 weeks. If therapeutic response (Hgb increase of >1g/dL or >3% increase in Hct) iron supplementation should continue for 2-3 months (2-3mg/kg/day). Hematologic and iron status should be rechecked 6 months after iron supplements are stopped to determine resolution and adequacy of iron stores. Nutritional counseling—prevention a. Maintain breastfeeding for 6 months if possible; supplemental iron drops or iron fortified cereal by 4 to 5 months of age b. If not breastfed, use iron fortified infant formula until 1 year of age c. Use iron fortified cereals from 6 to 12 months of age d. No cow's milk before 1 year of age, then limit to 18 to 24 oz/day e. Prescribe 2 to 3 mg/kg/day elemental iron in 1 to 2 doses/day for prophylaxis in low birth weight infants Failure to respond—consider the following reasons in this order (1) Failure or inconsistent administration of medication (2) Persistent or unrecognized blood loss (3) Incorrect diagnosis (4) Impaired GI absorption

Anemia of Infectious Disease a. Define each term b. Describe the pathophysiology c. List the clinical manifestations d. Discuss, briefly, the evaluation and management.

couldn't find this in book but found some info on uptodate a. Parvovirus is the infectious agent most commonly associated with acquired red cell aplasia; however, other infectious agents have been described (including hepatitis A, B, and C, Epstein-Barr virus, and HIV, malaria, tuberculsosis) immunocompromised patients are at risk Fetuses infected in utero may develop hydrops fetalis, with parvovirus being responsible for 10 to 15 percent of all cases of nonimmune hydrops b. Human parvovirus B19 is a common infection that results in erythema infectiosum (fifth disease). The virus has been associated with aplastic crisis in patients with hemolytic anemias, prolonged red cell aplasia and chronic anemias in patients with underlying immunodeficiency states, transient aplasia in the normal host. Parvovirus replicates solely in erythroid progenitor cells that have the P antigen receptor. individuals who are P antigen negative are resistant to the virus infection impairs RBC production, inadequate marrow response can be normocytic, normochromic anemia c. normal RBC turnover - no symptoms those with hemolytic anemia already - severe anemia or "aplastic (erythroblastopenic) crisis" heart failure Leukocytosis (high total white blood cell count) suggests infectious etiology Thrombocytosis commonly occurs as part of the acute phase reaction in response to infection d. may require tranfusion serum titers for parvovirus IgM and IgG, detection of viral DNA via polymerase chain reaction techniques on bone marrow samples, and routine bone marrow examination display giant proerythroblasts in their bone marrow samples decreased to absent erythroid precursors. may have pancytopenia Nonspecific tests: ESR Acute phase reactants: CRP, fibrinogen, haptoglobin serum ferritin serum iron and TIBC % iron saturation Bone marrow iron stores Bone marrow sideroblasts treat underlying disease of condition, anemia Most children who are healthy do not require treatment for parvovirus-related red cell aplasia. Patients with underlying chronic hemolysis (eg, sickle cell disease) who present with aplasia generally have resolution of their anemia in a short period of time and may only require transfusions based upon clinical symptoms caused by the anemia. Immunosuppressed patients may benefit from therapy with intravenous immune globulin (IVIG), which may result in a brisk reticulocytosis and rise in Hgb levels. Patients with continued immunosuppression, especially those with HIV, should be monitored for recurrence of the infection and may benefit from maintenance IVIG infusions

Congenital Hypercoagulability and Thrombosis a. Define each term b. Describe the pathophysiology c. List the clinical manifestations d. Discuss, briefly, the evaluation and management.

i couldn't find anything so this is from up to date and medscape a. Inherited thrombophilia is a genetic tendency to venous thromboembolism. The most frequent causes of an inherited (primary) hypercoagulable state are the factor V Leiden mutation and the prothrombin gene mutation. Defects in protein S, protein C, and antithrombin (formerly known as antithrombin III) account for most of the remaining cases. Heparin cofactor II deficiency — Heparin cofactor II (antithrombin is heparin cofactor I) is a heparin-dependent glycoprotein that acts as a thrombin inhibitor. Several families have been described with quantitative deficiency in this protein inherited as an autosomal dominant trait. Plasminogen deficiency — Congenital plasminogen deficiency is usually inherited as an autosomal dominant trait. Dysfibrinogenemia — Patients with dysfibrinogenemia have structural defects that cause alterations in the conversion of fibrinogen to fibrin. The most common structural defects involve the fibrinopeptides and their cleavage sites, and the second most common involves the gamma-chain polymerization region. Approximately one-half of the mutants are clinically silent, whereas hemorrhage and thrombosis occur in almost equal numbers of symptomatic patients. Patients with acquired hypercoagulable states or hereditary thrombophilia are more likely to develop clots, venous thrombosis, and arterial thrombosis b. The Virchow triad identifies the three underlying factors that are thought to contribute to thrombosis: hypercoagulability, hemodynamic dysfunction, and endothelial injury/dysfunction. Hypercoagulability can result from the release of procoagulants. Insufficient inactivation of procoagulants due to impaired regulatory antithrombotic pathways can result in hypercoagulability. The presence of factor V Leiden or a mutant prothrombin can cause hypercoagulability. The neutralization of activated factor Xa and thrombin are impaired in antithrombin (AT) deficiency. The formation of activated protein C (APC), which is a key down-regulator of factor V and factor VIII, may be impaired by protein C deficiency or protein S deficiency. Such deficiencies may be hereditary or acquired. [2] The ability of APC to inactivate factor V and factor VII can be impaired in individuals with mutant factor V such as factor V Leiden. This is known as APC resistance. Individuals with a mutant prothrombin (variously termed prothrombin G20210A, prothrombin G2010A, and mutant factor II) generate excess prothrombin that is associated with hypercoagulability. Injury to endothelium is accompanied by loss of protective molecules and expression of adhesive molecules, procoagulant activity, and mitogenic factors, leading to development of thrombosis, smooth muscle cell migration, and proliferation and atherosclerosis. c. no specific clinical symptoms or signs directly attributable to acquired hypercoagulability or hereditary thrombophilic disorders. venous thrombosis and pulmonary embolism. Hereditary thrombophilia should be suspected in patients with a history of any of the following: Recurrent venous thromboembolism Venous thrombosis before age 40 years A family history of venous thromboembolism Thrombosis in unusual sites (eg, mesenteric vein, renal vein, hepatic, or cerebral thrombosis) An association between hypercoagulability and severe obstructive sleep apnea has been reported d. ●Complete blood count with smear ●Routine coagulation studies ●Erythrocyte sedimentation rate (ESR) − A markedly elevated ESR (eg, >100 mm/hour) may suggest an underlying connective tissue disorder. ●Hemoccult stool ●Serum chemistries including liver and renal function tests and urinalysis ●Imaging - lower extremity ultrasound, computed tomographic pulmonary angiogram •Multiple thromboses or thrombosis in unusual organs (eg, mesenteric, hepatic, portal, splenic, renal, and cerebral veins) may suggest inherited thrombophilia - tested for all five inherited thrombophilias (levels of protein S, protein C, and antithrombin, the factor V Leiden and prothrombin gene mutations). Common tests in inheritable thrombophilia panels include the following: ●APC resistance/Factor V Leiden - activated protein C (APC) resistance is measured using a second-generation coagulation assay. Patients with low APC resistance ratios should then be genotyped for the mutation. ●Prothrombin gene mutation - The prothrombin 20210A gene mutation is best detected by molecular analysis. ●Proteins C and S - The best screening test for protein C deficiency is a functional assay which detects both quantitative and qualitative defects. The best screening test for protein S deficiency is the free protein S antigen assay. ●Antithrombin - The antithrombin-heparin (AT) cofactor assay using factor Xa or thrombin detects all currently recognized subtypes of familial AT deficiency. Direct factor Xa inhibitors (rivaroxaban, apixaban and edoxaban) and direct thrombin inhibitors (dabigatran), especially oral agents, have been developed as alternatives to warfarin. A hematologist experienced in the diagnosis and management of thrombophilias and hypercoagulable disorders should be consulted. Equally important is that the laboratory evaluations for thrombophilia are carried out in laboratories with extensive experience with these tests.

Glucose-6-Phosphate Dehydrogenase Deficiency (G6PD) a. Define each term b. Describe the pathophysiology c. List the clinical manifestations d. Discuss, briefly, the evaluation and management.

inherited a. common in African and Mediterranean descent, a drug induced hemolytic anemia caused by genetic deficiency of the G6PD enzyme in the RBC. Symptoms are generally associated with infections or exposure to oxidant metabolites of certain drugs that cause precipitation of Hgb, injury to the red cells, and rapid hemolysis. b. enzymopathy. The G6PD gene is found on the X chromosome. G6PD deficiency is transmitted as an X-linked autosomal recessive trait. Lack of G-6-PD decreases the ability to deal with oxidative stress and results in hemolysis; c. history of recent infection (particularly hepatitis) or oxidant drug ingestion—aspirin-containing antipyretics, sulfonamides, antimalarials, antihelmintics, naphthaquinolones, and fava beans. The degree of hemolysis is dependent on the amount of the drug ingested and the extent of enzyme deficiency. pallor and jaundice if there is chronic hemolysis, or have jaundice, pallor, lethargy, weakness, irritability, headache, and red or dark clear urine after drug ingestion. d. Several dye reduction tests provide the diagnosis. Screening tests should be used in high-risk groups. Only a few states include G6PD in their routine newborn screening panel. These tests measure G6PD enzyme activity in the RBC. After a hemolytic crisis screening may produce a false-negative result because the younger blood cells that remain after hemolysis may show normal enzymatic activity. This is thought to be associated with higher G6PD activity taking place in reticulocytes. The enzyme assay should be obtained 2 to 3 months after an episode No specific treatment is available. Red cell transfusion and supportive therapy may be indicated in cases of severe anemia. Keeping the child well hydrated and monitoring for renal failure are important during hemolytic crisis.

Autoimmune Neonatal Thrombocytopenias a. Define each term b. Describe the pathophysiology c. List the clinical manifestations d. Discuss, briefly, the evaluation and management.

not in book - from up to date, but see ITP card a. platelet count less than 150,000/microL mild (platelet count 100,000 to 150,000/microL), moderate (50,000 to 99,000/microL), and severe (<50,000/microL and/or persistent thrombocytopenia can result in bleeding) Autoimmune thrombocytopenia is mediated by maternal antibodies that react with both maternal and fetal platelets. This occurs in maternal autoimmune disorders, including immune thrombocytopenia purpura (ITP) and systemic lupus erythematosus (SLE). The diagnosis usually is apparent from the mother's medical history and maternal thrombocytopenia. the platelet count of affected mothers may be normal after a splenectomy or if there is sufficient compensatory thrombopoiesis. Mothers of infants with unexplained neonatal thrombocytopenia should be investigated for the presence of an autoimmune disorder because neonatal thrombocytopenia can be the presenting sign Healthy women without a history of an autoimmune disorder sometimes develop gestational thrombocytopenia that usually is mild This condition may be a mild and transient form of ITP and is considered benign. Neonatal thrombocytopenia is rare b. Causes classified by several different methods including platelet size (ie, large, normal, and small), mode of acquisition (congenital or acquired), early (<72 hours of age) or late (≥72 hours of age) onset, gestational age (GA), or underlying pathologic mechanisms. ●Increased destruction including immune-mediated platelet destruction or consumption ●Decreased production in the bone marrow Increased platelet destruction is the most common mechanism. In some conditions (eg, bacterial sepsis) both increased destruction and suppressed bone marrow production contribute to thrombocytopenia. thrombocytopenia due to an underlying immunologic mechanism is a common cause of early-onset neonatal thrombocytopenia. placental crossing of maternal antibodies, which destroy neonatal platelets c. Affected infants typically are otherwise healthy. Clinical signs are consistent with moderate to severe thrombocytopenia, including petechiae, bruising, and bleeding. 90% of infants born to mothers with ITP have safe (>50,000/microL) or normal platelet counts. Thrombocytopenia, when it occurs, is usually moderately severe, often in the range of 20,000 to 50,000/microL, The following are risk factors associated with autoimmune neonatal thrombocytopenia: ●Maternal splenectomy ●The mother's platelet count has been less than 50,000/microL at some time during the pregnancy [35,36] ●An older sibling has had neonatal thrombocytopenia [36] Platelet counts of infants born to mothers with ITP often decrease sharply during the several days after birth. The nadir typically occurs between two and five days of age. Thus, affected infants need to be closely monitored. platelet counts are obtained at a minimum daily and more frequently if there are clinical concerns for bleeding or the counts are below 50,000/microL. Once a trend showing an increase in platelet count is observed, the interval between testing is increased. The infant may be discharged, with appropriate outpatient follow-up, if the infant is asymptomatic once the platelet counts are stable, not decreasing, and above 30,000/microL. Thrombocytopenia appears to be due to the transfer of antiplatelet antibodies of the immunoglobulin A (IgA) type in the milk of affected mothers and resolved when breastfeeding was discontinued. d. well-appearing term infant who presents with severe unexplained thrombocytopenia in the first 24 to 48 hours of life. also needs to be considered in ill-appearing infants, especially if severe thrombocytopenia is present, appears to be out of proportion to the clinical illness, or persists when the clinical illness improves. serologic testing that demonstrates the presence of maternal antihuman platelet antibody In the first affected child, antigen testing of the mother's and father's platelets should be performed, and the mother's serum should be tested for antiplatelet alloantibodies. If available, DNA testing can be performed on the maternal and paternal blood to identify the platelet antigen genotypes. ●initial evaluation - cranial ultrasound examination to detect hemorrhage. If an intracranial hemorrhage is present, a higher threshold should be used for platelet transfusion (<50,000/microL). If a platelet transfusion is administered, the platelet count should be maintained above 50,000/microL or 100,000/microL in infants who have evidence of intracranial hemorrhage A platelet transfusion is given to infants with very severe thrombocytopenia or clinical bleeding. transfusions may not be as effective as in infants with thrombocytopenia due to other causes because the autoantibodies usually will react with all donor platelets, including those of the mother. IVIG should be administered to infants with severe thrombocytopenia at a dose of 1 g/kg (repeated if necessary), which typically produces a rapid response. Neonatal thrombocytopenia secondary to maternal ITP may last for weeks to months and require long-term monitoring. Occasionally, a second dose of IVIG is required at four to six weeks after birth. If thrombocytopenia is severe and persistent after IVIG therapy, some clinicians give a short course of prednisone (2 mg/kg per day) or methylprednisolone (1 mg/kg twice a day by mouth for five days) [29].

Autoimmune Vascular Purpura a. Define each term b. Describe the pathophysiology c. List the clinical manifestations d. Discuss, briefly, the evaluation and management.

not in book, from up to date a. Bleeding under the skin or into mucosal membranes is called purpura and may be an innocent finding related to mild childhood trauma or may be the presenting sign of a life threatening disease. Purpura can be subdivided based on size into petechiae and ecchymoses Pinpoint areas (less than 2 mm) of hemorrhage, which are reddish-purple lesions are called petechiae, while larger confluent lesions are referred to as ecchymoses. Ecchymoses are commonly called bruises. In some cases, ecchymoses may be tender or raised. In contrast to other erythematous or vascular skin lesions, purpuric lesions do not blanch when pressure is applied to the skin. b. Purpura may result from disruptions in vascular integrity or may be due to abnormalities in primary or secondary hemostasis. ●Vitamin C and other factors that affect collagen synthesis are required for normal formation of connective tissue within the vessel walls. When a blood vessel is injured, vasoconstriction and retraction usually occur immediately and decrease the flow of blood to the affected area. ●Formation of a platelet plug, also referred to as primary hemostasis, is initiated upon injury to a blood vessel wall ●The platelet plug alone is not sufficient, and requires the second phase of hemostasis to form a stable fibrin clot. tissue factor binds to factor VII. cascading reactions whose central purpose is to generate thrombin from prothrombin (II) at the site of injury. Thrombin can then cleave fibrinogen to fibrin, which is cross-linked and forms a clot. Immunoglobulin A vasculitis (IgAV; Henoch-Schönlein purpura [HSP]) — IgAV (HSP) is the most common cause of vasculitis in children, occurring at a mean age of six years. The clinical manifestations of IgAV (HSP)are due to IgA1 deposition in blood vessel walls and the renal mesangium. Cutaneous purpura is an essential component in the diagnosis of IgAV (HSP) and is characterized by palpable purpuric lesions 2 to 10 mm in diameter that often coalesce. The purpura tend to be concentrated on the buttocks and lower extremities. Other clinical manifestations of IgAV (HSP) include arthritis (commonly of the knees and ankles), abdominal pain with or without gastrointestinal bleeding, and nephritis. Neonatal purpura fulminans — usually occurs on the first day of life and is caused by congenital severe deficiency of protein C or protein S. Affected infants present with ecchymoses, extensive venous and arterial thromboses, laboratory evidence of disseminated intravascular coagulation, and extremely low levels of protein C antigen. Administration of exogenous protein replacement (Protein C concentrate or fresh frozen plasma) appears to be critical for the treatment of neonatal purpura fulminans, while heparin and antiplatelet agents are ineffective. Severe acquired deficiency of the anticoagulant proteins, protein C and protein S, may also present with non-infectious purpura fulminans or in association with infectious disseminated intravascular coagulopathy. Ehlers-Danlos syndrome — group of inherited disorders that involve genetic defects in collagen and connective-tissue synthesis and structure. This heterogeneous group is characterized by joint hypermobility, cutaneous fragility, and hyperextensibility. Fragility of dermal skin is common, with frequent bruises and delayed wound healing. In some cases, abnormal blood vessel structure causes capillary hemorrhage. Pigmented purpuric dermatoses (PPDs) — capillaritis, purpura simplex, and inflammatory purpura without vasculitis, chronic, benign, cutaneous eruptions characterized by the presence of petechiae, purpura, and increased skin pigmentation. most commonly occur on the lower extremities and may be asymptomatic or pruritic. The presence of discrete or circumscribed purpuric macules, papules, patches, or plaques (particularly on the lower extremity) with petechiae and yellow-brown pigmentation suggests the possibility of PPDs. The diagnosis is usually made through clinical inspection and the recognition of classic clinical features. A skin biopsy is useful when the diagnosis remains uncertain following clinical examination. Biopsies may help to distinguish PPD from disorders in the differential diagnosis, such as cutaneous vasculitis. c.●Fever - Serious infection is an important cause of purpura in children and is usually associated with fever. ●Age of patient - Severe congenital bleeding disorders usually present in the first year of life. ●Sex of patient - Hemophilia is X-linked and only affects males. Von Willebrand is autosomal dominant and is seen with equal frequency in boys and girls. ●Onset of symptoms - recent and past medical history should be reviewed carefully with the parents and child. Acute onset of purpura after a recent viral illness or immunization is consistent with an acquired disorder such as ITP. Recurrent purpura since infancy, however, suggests an inherited abnormality of platelets or clotting factors. ●Location and type of purpura - Petechiae are most commonly seen in patients with thrombocytopenia and platelet disorders, mucosal bleeding (menorrhagia, epistaxis) is common in platelet disorders and von Willebrand disease, and bleeding into the joints (hemarthroses) is common in patients with hemophilia. ●Prior bleeding history - Specific inquiries about past surgeries (including circumcision), dental extractions, or significant trauma should be made because the absence of bleeding under these conditions would be unusual in most inherited disorders of even moderate severity. ●Family history of bleeding ●Recent medications - exposure to aspirin and other drugs known to affect platelet function. ●Exploratory or intentional overdose of prescription anticoagulant or rat poison (warfarin or superwarfarin) ingestion ●Dietary history - Patients with severe malnutrition or restricted diets may be at risk for scurvy or vitamin K deficiency. Physical examination ●Skin - red, blue, or purple bruises suggest recent lesions. Yellow, brown, or green bruises tend to be older, healing lesions. Ecchymotic lesions may take weeks to resolve. Petechiae indicate disorders associated with thrombocytopenia or platelet dysfunction. Clotting disorders typically cause mucosal bleeding or ecchymoses without petechiae Bruising in a recognizable pattern, such as a loop mark, ligature mark, belt or hand print, indicates child abuse. Bruises in multiple stages of healing in areas atypical for unintentional trauma are also suspicious Purpura localized to the lower body (buttocks, legs, ankles), with or without joint swelling in the hands or feet, suggests immunoglobulin A vasculitis (IgAV; Henoch-Schönlein purpura [HSP]). ●Lymph nodes - Lymphadenopathy may be present in certain malignancies (leukemias) or viral infections (infectious mononucleosis, CMV) that can present with purpura. ●Abdominal examination - Hepatomegaly may signal an underlying hepatic disorder or leukemia. Splenomegaly can be seen in infectious mononucleosis, leukemia, hepatic disease, and the storage diseases. ●Extremities and joints - Inflammation or synovial thickening of the large joints is consistent with the hemarthroses seen in hemophilia or the swollen, painful joints associated with IgAV (HSP). ●Neurologic evaluation - Complete neurologic assessment is mandatory when there is suspicion of head trauma in the face of a bleeding diathesis. The eyes should be examined for the presence of conjunctival, scleral, or retinal hemorrhage. d. ●Complete blood count (CBC), including a platelet count and a peripheral smear ●Prothrombin time (PT) with an international normalized ratio (INR) ●Activated partial thromboplastin time (aPTT) The bleeding time is not routinely recommended in children. ●Thrombocytopenia: •Inspection of blood smear (screening for bone marrow diseases, platelet size and platelet clumping) •Mean platelet volume (elevated in destructive causes, low in Wiskott-Aldrich syndrome) •Bone marrow aspiration (in cases where malignancy or bone marrow failure is suspected) ●PT and aPTT prolonged and thrombocytopenia (suspect DIC): •Fibrinogen •D-dimer or fibrin split products •Peripheral smear inspection for RBC fragments (schistocytes) ●Prolonged PT and aPTT with normal platelet count: •Factor assays: II (prothrombin), V, VII, VIII and fibrinogen to help distinguish between hepatic dysfunction and vitamin K deficiency •Hepatic enzymes and serum albumin to assess hepatic function ●Prolonged aPTT only: •Inhibitor screen (50:50 mixing study of patient's and normal plasma) •If aPTT fully corrects: Specific factors: VIII, IX, XI, XII, VW Ag, VW Activity •If partial or no correction after mixing study: Inhibitor is present Confirmatory test for the presence of a lupus anticoagulant ●Prolonged PT only: •Factor VII level ●Initial CBC, smear, PT, and aPTT normal: •Qualitative platelet defect suspected. consider platelet aggregometry. •von Willebrand disease (VWD) suspected. obtain factor VIII, von Willebrand factor antigen and von Willebrand factor activity. If the VW antigen or activity are low, vWf multimeric analysis should be performed to aid in subtype determination. The normal ranges of these tests depend on the patient's ABO blood group type; values fluctuate over time and may periodically be normal in affected individuals. Appears ill — Children with purpura who are ill-appearing or febrile require rapid evaluation and treatment for serious hemorrhage, disseminated intravascular coagulopathy, or infection. Patients with abnormal vital signs warrant emergent attention and stabilization of the airway, breathing, and circulation. target likely underlying etiologies including serious trauma, child abuse, and bacterial sepsis (meningococcemia). Empiric antibiotic treatment is usually warranted. Fibrinogen, D-dimer or fibrin split products (FSP), peripheral smear, and liver testing (albumin and liver enzymes) should be performed in addition to screening coagulation studies (CBC, PT, aPTT). Appears well — Well-appearing children with purpura should undergo a detailed history, physical examination, and screening coagulation studies (CBC, PT, aPTT) to establish the most likely diagnosis.

Insulin Types and Profiles (ppt)

rapid acting analogues: bolus insulin insulin lispro (Humalog) onset of action (ooa) (h) 1/4-1/2, peak action (pa) 1/2- 1 1/4, effective duration of action (edoa) 3-4, maximum duration (md) 4-6 insulin aspart (NovoLog): ooa: 1/4-1/2, pa: 1/2 - 1 1/4, edoa: 3-4, md: 4-6 insulin glulisine (Apidra): ooa: 1/4-1/2, pa: 1/2 1 1/4, edoa: 3-4, md: 4-6 short acting: used for insulin drip, can be used as bolus insulin regular (soluble): oa: 1/2-1, pa: 2-3, edoa: 3-6, md: 6-8 intermediate acting: infrequently used, can be used as "basal" given BID NPH (isophane): ooa: 2-4, pa: 6-10, edoa: 10-16, md: 14-18 long-acting analogue: basal insulin insulin glargine (Lantus): ooa: 3-4, pa: 8-16, edoa: 18-20, md: 20-24 insulin detemir (Levemir): ooa: 3-4, pa: 6-8 (though relatively flat), edaa: 14, md: up to 20-24 When converting from NPH to glargine, glargine dose should be 80% of total NPH dose notice that detemir has a slight peak and potentially shorter duration


Related study sets

Chapter 12: Small Business Accounting: Projecting and Evaluating PerformanceAssignment

View Set

6.6: Regulating Blood Glucose Levels

View Set

Which of the following statements is true regarding salespeople?

View Set

Prep U Chapter 6: Nursing Care for the Family in Need of Reproductive Life Planning

View Set

Proverbs 27 - Flashcard MC Questions - Ted Hildebrandt

View Set

Abeka Grade 7 History of the World Quiz 34

View Set

Chapter 8 and 9 Test Multiple Choice

View Set

Mythology Michigan Virtual Test Answers

View Set

phy 112 midterm practice problems

View Set